Why isn't the quotient space $X=mathbbR^2/(mathbbR times 0)$ first-countable? The Next CEO of Stack OverflowSequential compactness vs. countable compactnessThe Line with Uncountably Many Origins: Second Countable?Why isn't the half-disk topology separable?Do spaces satisfying the first axiom of countability have monotone decreasing bases for every point?Hausdorff Non-First-Countable Quotient Spaces of Ordered SquareA quotient space that does not satisfy the first countability axiomSecond Countable, First Countable, and Separable SpacesWhat is a first countable, limit compact space that is not sequentially compact?A first countable, countably compact space is sequentially compactA first countable hemicompact space is locally compact

Unreliable Magic - Is it worth it?

Anatomically Correct Mesopelagic Aves

The King's new dress

What does "Its cash flow is deeply negative" mean?

MAZDA 3 2006 (UK) - poor acceleration then takes off at 3250 revs

Is a stroke of luck acceptable after a series of unfavorable events?

What is the difference between "behavior" and "behaviour"?

Horror movie/show or scene where a horse creature opens its mouth really wide and devours a man in a stables

Is it my responsibility to learn a new technology in my own time my employer wants to implement?

How can I quit an app using Terminal?

Describing a person. What needs to be mentioned?

How to count occurrences of text in a file?

Robert Sheckley short story about vacation spots being overwhelmed

What's the point of interval inversion?

Why do remote companies require working in the US?

Return the Closest Prime Number

Should I tutor a student who I know has cheated on their homework?

Would this house-rule that treats advantage as a +1 to the roll instead (and disadvantage as -1) and allows them to stack be balanced?

Are there languages with no euphemisms?

What happens if you roll doubles 3 times then land on "Go to jail?"

Is HostGator storing my password in plaintext?

Why were Madagascar and New Zealand discovered so late?

How can I get through very long and very dry, but also very useful technical documents when learning a new tool?

What is the point of a new vote on May's deal when the indicative votes suggest she will not win?



Why isn't the quotient space $X=mathbbR^2/(mathbbR times 0)$ first-countable?



The Next CEO of Stack OverflowSequential compactness vs. countable compactnessThe Line with Uncountably Many Origins: Second Countable?Why isn't the half-disk topology separable?Do spaces satisfying the first axiom of countability have monotone decreasing bases for every point?Hausdorff Non-First-Countable Quotient Spaces of Ordered SquareA quotient space that does not satisfy the first countability axiomSecond Countable, First Countable, and Separable SpacesWhat is a first countable, limit compact space that is not sequentially compact?A first countable, countably compact space is sequentially compactA first countable hemicompact space is locally compact










4












$begingroup$


Consider the quotient space $X=mathbbR^2/(mathbbR times 0)$ I'm supposed to prove it isn't first countable. It seems to me that this space is homeomorphic to two open triangles joined at their apexes with that point added. That point is the problematic one, but why isn't the collection of open balls with descending rational radii centered at this point intersected with $X$ a countable basis?



Thank you for your answers.



Edit: I found this problem on my university's website. We did this same problem at school but first-countability was replaced with local compactness. It could be possible that the space is in fact first-countable, and the problem statement on the website was incorrect.










share|cite|improve this question











$endgroup$







  • 1




    $begingroup$
    Consider the neighbourhood $(x, y)in Bbb R^2mid -e^-x^2<y<e^-x^2$ of the problematic point. That's not contained in any of your rational-radius neighbourhoods. So that's a counter to your one example. Haven't figured out a general proof, though.
    $endgroup$
    – Arthur
    yesterday











  • $begingroup$
    Maybe we can use diagonalization? Observe your statement is equivalent to: There doesn't exist a countable collection of open sets $U_n$ such that for every open set $U$ containing $mathbb Rtimes 0$, $mathbb Rtimes 0subset U_nsubset U$ for some $n$.
    $endgroup$
    – YuiTo Cheng
    yesterday











  • $begingroup$
    @Arthur, I thought it was the other way around, i. e. that any open set containing the point has to contain a basis element?
    $endgroup$
    – InsertNameHere
    yesterday







  • 1




    $begingroup$
    @InsertNameHere You're right. I didn't keep my phrasing straight. Still, my point stands: that set is a witness that your proposed basis isn't actually a basis.
    $endgroup$
    – Arthur
    yesterday











  • $begingroup$
    What is the equivalence relation being used to get the quotient space?
    $endgroup$
    – Nemo
    yesterday















4












$begingroup$


Consider the quotient space $X=mathbbR^2/(mathbbR times 0)$ I'm supposed to prove it isn't first countable. It seems to me that this space is homeomorphic to two open triangles joined at their apexes with that point added. That point is the problematic one, but why isn't the collection of open balls with descending rational radii centered at this point intersected with $X$ a countable basis?



Thank you for your answers.



Edit: I found this problem on my university's website. We did this same problem at school but first-countability was replaced with local compactness. It could be possible that the space is in fact first-countable, and the problem statement on the website was incorrect.










share|cite|improve this question











$endgroup$







  • 1




    $begingroup$
    Consider the neighbourhood $(x, y)in Bbb R^2mid -e^-x^2<y<e^-x^2$ of the problematic point. That's not contained in any of your rational-radius neighbourhoods. So that's a counter to your one example. Haven't figured out a general proof, though.
    $endgroup$
    – Arthur
    yesterday











  • $begingroup$
    Maybe we can use diagonalization? Observe your statement is equivalent to: There doesn't exist a countable collection of open sets $U_n$ such that for every open set $U$ containing $mathbb Rtimes 0$, $mathbb Rtimes 0subset U_nsubset U$ for some $n$.
    $endgroup$
    – YuiTo Cheng
    yesterday











  • $begingroup$
    @Arthur, I thought it was the other way around, i. e. that any open set containing the point has to contain a basis element?
    $endgroup$
    – InsertNameHere
    yesterday







  • 1




    $begingroup$
    @InsertNameHere You're right. I didn't keep my phrasing straight. Still, my point stands: that set is a witness that your proposed basis isn't actually a basis.
    $endgroup$
    – Arthur
    yesterday











  • $begingroup$
    What is the equivalence relation being used to get the quotient space?
    $endgroup$
    – Nemo
    yesterday













4












4








4


2



$begingroup$


Consider the quotient space $X=mathbbR^2/(mathbbR times 0)$ I'm supposed to prove it isn't first countable. It seems to me that this space is homeomorphic to two open triangles joined at their apexes with that point added. That point is the problematic one, but why isn't the collection of open balls with descending rational radii centered at this point intersected with $X$ a countable basis?



Thank you for your answers.



Edit: I found this problem on my university's website. We did this same problem at school but first-countability was replaced with local compactness. It could be possible that the space is in fact first-countable, and the problem statement on the website was incorrect.










share|cite|improve this question











$endgroup$




Consider the quotient space $X=mathbbR^2/(mathbbR times 0)$ I'm supposed to prove it isn't first countable. It seems to me that this space is homeomorphic to two open triangles joined at their apexes with that point added. That point is the problematic one, but why isn't the collection of open balls with descending rational radii centered at this point intersected with $X$ a countable basis?



Thank you for your answers.



Edit: I found this problem on my university's website. We did this same problem at school but first-countability was replaced with local compactness. It could be possible that the space is in fact first-countable, and the problem statement on the website was incorrect.







general-topology






share|cite|improve this question















share|cite|improve this question













share|cite|improve this question




share|cite|improve this question








edited yesterday







InsertNameHere

















asked yesterday









InsertNameHereInsertNameHere

773




773







  • 1




    $begingroup$
    Consider the neighbourhood $(x, y)in Bbb R^2mid -e^-x^2<y<e^-x^2$ of the problematic point. That's not contained in any of your rational-radius neighbourhoods. So that's a counter to your one example. Haven't figured out a general proof, though.
    $endgroup$
    – Arthur
    yesterday











  • $begingroup$
    Maybe we can use diagonalization? Observe your statement is equivalent to: There doesn't exist a countable collection of open sets $U_n$ such that for every open set $U$ containing $mathbb Rtimes 0$, $mathbb Rtimes 0subset U_nsubset U$ for some $n$.
    $endgroup$
    – YuiTo Cheng
    yesterday











  • $begingroup$
    @Arthur, I thought it was the other way around, i. e. that any open set containing the point has to contain a basis element?
    $endgroup$
    – InsertNameHere
    yesterday







  • 1




    $begingroup$
    @InsertNameHere You're right. I didn't keep my phrasing straight. Still, my point stands: that set is a witness that your proposed basis isn't actually a basis.
    $endgroup$
    – Arthur
    yesterday











  • $begingroup$
    What is the equivalence relation being used to get the quotient space?
    $endgroup$
    – Nemo
    yesterday












  • 1




    $begingroup$
    Consider the neighbourhood $(x, y)in Bbb R^2mid -e^-x^2<y<e^-x^2$ of the problematic point. That's not contained in any of your rational-radius neighbourhoods. So that's a counter to your one example. Haven't figured out a general proof, though.
    $endgroup$
    – Arthur
    yesterday











  • $begingroup$
    Maybe we can use diagonalization? Observe your statement is equivalent to: There doesn't exist a countable collection of open sets $U_n$ such that for every open set $U$ containing $mathbb Rtimes 0$, $mathbb Rtimes 0subset U_nsubset U$ for some $n$.
    $endgroup$
    – YuiTo Cheng
    yesterday











  • $begingroup$
    @Arthur, I thought it was the other way around, i. e. that any open set containing the point has to contain a basis element?
    $endgroup$
    – InsertNameHere
    yesterday







  • 1




    $begingroup$
    @InsertNameHere You're right. I didn't keep my phrasing straight. Still, my point stands: that set is a witness that your proposed basis isn't actually a basis.
    $endgroup$
    – Arthur
    yesterday











  • $begingroup$
    What is the equivalence relation being used to get the quotient space?
    $endgroup$
    – Nemo
    yesterday







1




1




$begingroup$
Consider the neighbourhood $(x, y)in Bbb R^2mid -e^-x^2<y<e^-x^2$ of the problematic point. That's not contained in any of your rational-radius neighbourhoods. So that's a counter to your one example. Haven't figured out a general proof, though.
$endgroup$
– Arthur
yesterday





$begingroup$
Consider the neighbourhood $(x, y)in Bbb R^2mid -e^-x^2<y<e^-x^2$ of the problematic point. That's not contained in any of your rational-radius neighbourhoods. So that's a counter to your one example. Haven't figured out a general proof, though.
$endgroup$
– Arthur
yesterday













$begingroup$
Maybe we can use diagonalization? Observe your statement is equivalent to: There doesn't exist a countable collection of open sets $U_n$ such that for every open set $U$ containing $mathbb Rtimes 0$, $mathbb Rtimes 0subset U_nsubset U$ for some $n$.
$endgroup$
– YuiTo Cheng
yesterday





$begingroup$
Maybe we can use diagonalization? Observe your statement is equivalent to: There doesn't exist a countable collection of open sets $U_n$ such that for every open set $U$ containing $mathbb Rtimes 0$, $mathbb Rtimes 0subset U_nsubset U$ for some $n$.
$endgroup$
– YuiTo Cheng
yesterday













$begingroup$
@Arthur, I thought it was the other way around, i. e. that any open set containing the point has to contain a basis element?
$endgroup$
– InsertNameHere
yesterday





$begingroup$
@Arthur, I thought it was the other way around, i. e. that any open set containing the point has to contain a basis element?
$endgroup$
– InsertNameHere
yesterday





1




1




$begingroup$
@InsertNameHere You're right. I didn't keep my phrasing straight. Still, my point stands: that set is a witness that your proposed basis isn't actually a basis.
$endgroup$
– Arthur
yesterday





$begingroup$
@InsertNameHere You're right. I didn't keep my phrasing straight. Still, my point stands: that set is a witness that your proposed basis isn't actually a basis.
$endgroup$
– Arthur
yesterday













$begingroup$
What is the equivalence relation being used to get the quotient space?
$endgroup$
– Nemo
yesterday




$begingroup$
What is the equivalence relation being used to get the quotient space?
$endgroup$
– Nemo
yesterday










1 Answer
1






active

oldest

votes


















2












$begingroup$

$mathbbR times 0$ is collapsed to a point $ast in X=mathbbR^2/(mathbbR times 0)$. This point does not have a countable base of open neighborhoods.



$X$ is endowed with the quotient topology. This means that open neigborhoods of $ast$ are in a $1$-$1$-correspondence with open subsets of $mathbbR^2$ containing $mathbbR times 0$. Let us denote the collection of these sets by $mathfrakU$.



Therefore, as YuiTo Cheng remarked in his comments, we have to show that there does not exist a countable collection of sets $U_ n in mathfrakU$ such that for every $U in mathfrakU$, $U_n subset U$ for some $n$.



Let us consider any countable collection of $U_ n in mathfrakU$. For each $n in mathbbN$ there exists a number $u_n > 0$ such that $(n,u_n) in U_n$. The set $D = (n,u_n) mid n in mathbbN $ is closed in $mathbbR^2$, thus $U = mathbbR^2 setminus D in mathfrakU$. But by construction no $U_n$ can be contained in $U$.



Remark:



This generalizes as follows. Let $X,Y$ be topological spaces such that $X$ contains an infinite subset $A subset X$ without limit points and let $y in Y$ be a point such that $ y $ is not open in $Y$. Then $(X times Y)/(X times y )$ is not first countable.






share|cite|improve this answer











$endgroup$













    Your Answer





    StackExchange.ifUsing("editor", function ()
    return StackExchange.using("mathjaxEditing", function ()
    StackExchange.MarkdownEditor.creationCallbacks.add(function (editor, postfix)
    StackExchange.mathjaxEditing.prepareWmdForMathJax(editor, postfix, [["$", "$"], ["\\(","\\)"]]);
    );
    );
    , "mathjax-editing");

    StackExchange.ready(function()
    var channelOptions =
    tags: "".split(" "),
    id: "69"
    ;
    initTagRenderer("".split(" "), "".split(" "), channelOptions);

    StackExchange.using("externalEditor", function()
    // Have to fire editor after snippets, if snippets enabled
    if (StackExchange.settings.snippets.snippetsEnabled)
    StackExchange.using("snippets", function()
    createEditor();
    );

    else
    createEditor();

    );

    function createEditor()
    StackExchange.prepareEditor(
    heartbeatType: 'answer',
    autoActivateHeartbeat: false,
    convertImagesToLinks: true,
    noModals: true,
    showLowRepImageUploadWarning: true,
    reputationToPostImages: 10,
    bindNavPrevention: true,
    postfix: "",
    imageUploader:
    brandingHtml: "Powered by u003ca class="icon-imgur-white" href="https://imgur.com/"u003eu003c/au003e",
    contentPolicyHtml: "User contributions licensed under u003ca href="https://creativecommons.org/licenses/by-sa/3.0/"u003ecc by-sa 3.0 with attribution requiredu003c/au003e u003ca href="https://stackoverflow.com/legal/content-policy"u003e(content policy)u003c/au003e",
    allowUrls: true
    ,
    noCode: true, onDemand: true,
    discardSelector: ".discard-answer"
    ,immediatelyShowMarkdownHelp:true
    );



    );













    draft saved

    draft discarded


















    StackExchange.ready(
    function ()
    StackExchange.openid.initPostLogin('.new-post-login', 'https%3a%2f%2fmath.stackexchange.com%2fquestions%2f3164388%2fwhy-isnt-the-quotient-space-x-mathbbr2-mathbbr-times-0-first-co%23new-answer', 'question_page');

    );

    Post as a guest















    Required, but never shown

























    1 Answer
    1






    active

    oldest

    votes








    1 Answer
    1






    active

    oldest

    votes









    active

    oldest

    votes






    active

    oldest

    votes









    2












    $begingroup$

    $mathbbR times 0$ is collapsed to a point $ast in X=mathbbR^2/(mathbbR times 0)$. This point does not have a countable base of open neighborhoods.



    $X$ is endowed with the quotient topology. This means that open neigborhoods of $ast$ are in a $1$-$1$-correspondence with open subsets of $mathbbR^2$ containing $mathbbR times 0$. Let us denote the collection of these sets by $mathfrakU$.



    Therefore, as YuiTo Cheng remarked in his comments, we have to show that there does not exist a countable collection of sets $U_ n in mathfrakU$ such that for every $U in mathfrakU$, $U_n subset U$ for some $n$.



    Let us consider any countable collection of $U_ n in mathfrakU$. For each $n in mathbbN$ there exists a number $u_n > 0$ such that $(n,u_n) in U_n$. The set $D = (n,u_n) mid n in mathbbN $ is closed in $mathbbR^2$, thus $U = mathbbR^2 setminus D in mathfrakU$. But by construction no $U_n$ can be contained in $U$.



    Remark:



    This generalizes as follows. Let $X,Y$ be topological spaces such that $X$ contains an infinite subset $A subset X$ without limit points and let $y in Y$ be a point such that $ y $ is not open in $Y$. Then $(X times Y)/(X times y )$ is not first countable.






    share|cite|improve this answer











    $endgroup$

















      2












      $begingroup$

      $mathbbR times 0$ is collapsed to a point $ast in X=mathbbR^2/(mathbbR times 0)$. This point does not have a countable base of open neighborhoods.



      $X$ is endowed with the quotient topology. This means that open neigborhoods of $ast$ are in a $1$-$1$-correspondence with open subsets of $mathbbR^2$ containing $mathbbR times 0$. Let us denote the collection of these sets by $mathfrakU$.



      Therefore, as YuiTo Cheng remarked in his comments, we have to show that there does not exist a countable collection of sets $U_ n in mathfrakU$ such that for every $U in mathfrakU$, $U_n subset U$ for some $n$.



      Let us consider any countable collection of $U_ n in mathfrakU$. For each $n in mathbbN$ there exists a number $u_n > 0$ such that $(n,u_n) in U_n$. The set $D = (n,u_n) mid n in mathbbN $ is closed in $mathbbR^2$, thus $U = mathbbR^2 setminus D in mathfrakU$. But by construction no $U_n$ can be contained in $U$.



      Remark:



      This generalizes as follows. Let $X,Y$ be topological spaces such that $X$ contains an infinite subset $A subset X$ without limit points and let $y in Y$ be a point such that $ y $ is not open in $Y$. Then $(X times Y)/(X times y )$ is not first countable.






      share|cite|improve this answer











      $endgroup$















        2












        2








        2





        $begingroup$

        $mathbbR times 0$ is collapsed to a point $ast in X=mathbbR^2/(mathbbR times 0)$. This point does not have a countable base of open neighborhoods.



        $X$ is endowed with the quotient topology. This means that open neigborhoods of $ast$ are in a $1$-$1$-correspondence with open subsets of $mathbbR^2$ containing $mathbbR times 0$. Let us denote the collection of these sets by $mathfrakU$.



        Therefore, as YuiTo Cheng remarked in his comments, we have to show that there does not exist a countable collection of sets $U_ n in mathfrakU$ such that for every $U in mathfrakU$, $U_n subset U$ for some $n$.



        Let us consider any countable collection of $U_ n in mathfrakU$. For each $n in mathbbN$ there exists a number $u_n > 0$ such that $(n,u_n) in U_n$. The set $D = (n,u_n) mid n in mathbbN $ is closed in $mathbbR^2$, thus $U = mathbbR^2 setminus D in mathfrakU$. But by construction no $U_n$ can be contained in $U$.



        Remark:



        This generalizes as follows. Let $X,Y$ be topological spaces such that $X$ contains an infinite subset $A subset X$ without limit points and let $y in Y$ be a point such that $ y $ is not open in $Y$. Then $(X times Y)/(X times y )$ is not first countable.






        share|cite|improve this answer











        $endgroup$



        $mathbbR times 0$ is collapsed to a point $ast in X=mathbbR^2/(mathbbR times 0)$. This point does not have a countable base of open neighborhoods.



        $X$ is endowed with the quotient topology. This means that open neigborhoods of $ast$ are in a $1$-$1$-correspondence with open subsets of $mathbbR^2$ containing $mathbbR times 0$. Let us denote the collection of these sets by $mathfrakU$.



        Therefore, as YuiTo Cheng remarked in his comments, we have to show that there does not exist a countable collection of sets $U_ n in mathfrakU$ such that for every $U in mathfrakU$, $U_n subset U$ for some $n$.



        Let us consider any countable collection of $U_ n in mathfrakU$. For each $n in mathbbN$ there exists a number $u_n > 0$ such that $(n,u_n) in U_n$. The set $D = (n,u_n) mid n in mathbbN $ is closed in $mathbbR^2$, thus $U = mathbbR^2 setminus D in mathfrakU$. But by construction no $U_n$ can be contained in $U$.



        Remark:



        This generalizes as follows. Let $X,Y$ be topological spaces such that $X$ contains an infinite subset $A subset X$ without limit points and let $y in Y$ be a point such that $ y $ is not open in $Y$. Then $(X times Y)/(X times y )$ is not first countable.







        share|cite|improve this answer














        share|cite|improve this answer



        share|cite|improve this answer








        edited yesterday

























        answered yesterday









        Paul FrostPaul Frost

        11.7k3935




        11.7k3935



























            draft saved

            draft discarded
















































            Thanks for contributing an answer to Mathematics Stack Exchange!


            • Please be sure to answer the question. Provide details and share your research!

            But avoid


            • Asking for help, clarification, or responding to other answers.

            • Making statements based on opinion; back them up with references or personal experience.

            Use MathJax to format equations. MathJax reference.


            To learn more, see our tips on writing great answers.




            draft saved


            draft discarded














            StackExchange.ready(
            function ()
            StackExchange.openid.initPostLogin('.new-post-login', 'https%3a%2f%2fmath.stackexchange.com%2fquestions%2f3164388%2fwhy-isnt-the-quotient-space-x-mathbbr2-mathbbr-times-0-first-co%23new-answer', 'question_page');

            );

            Post as a guest















            Required, but never shown





















































            Required, but never shown














            Required, but never shown












            Required, but never shown







            Required, but never shown

































            Required, but never shown














            Required, but never shown












            Required, but never shown







            Required, but never shown







            Popular posts from this blog

            Triangular numbers and gcdProving sum of a set is $0 pmod n$ if $n$ is odd, or $fracn2 pmod n$ if $n$ is even?Is greatest common divisor of two numbers really their smallest linear combination?GCD, LCM RelationshipProve a set of nonnegative integers with greatest common divisor 1 and closed under addition has all but finite many nonnegative integers.all pairs of a and b in an equation containing gcdTriangular Numbers Modulo $k$ - Hit All Values?Understanding the Existence and Uniqueness of the GCDGCD and LCM with logical symbolsThe greatest common divisor of two positive integers less than 100 is equal to 3. Their least common multiple is twelve times one of the integers.Suppose that for all integers $x$, $x|a$ and $x|b$ if and only if $x|c$. Then $c = gcd(a,b)$Which is the gcd of 2 numbers which are multiplied and the result is 600000?

            Ingelân Ynhâld Etymology | Geografy | Skiednis | Polityk en bestjoer | Ekonomy | Demografy | Kultuer | Klimaat | Sjoch ek | Keppelings om utens | Boarnen, noaten en referinsjes Navigaasjemenuwww.gov.ukOffisjele webside fan it regear fan it Feriene KeninkrykOffisjele webside fan it Britske FerkearsburoNederlânsktalige ynformaasje fan it Britske FerkearsburoOffisjele webside fan English Heritage, de organisaasje dy't him ynset foar it behâld fan it Ingelske kultuergoedYnwennertallen fan alle Britske stêden út 'e folkstelling fan 2011Notes en References, op dizze sideEngland

            Հադիս Բովանդակություն Անվանում և նշանակություն | Դասակարգում | Աղբյուրներ | Նավարկման ցանկ